What is the solution to the system of equations graphed below?

What Is The Solution To The System Of Equations Graphed Below?

Answers

Answer 1

the answer is C, hope this helps.

Answer 2

Answer:

BELOW

Step-by-step explanation:

(Y, X)

Y=-1

X=1

*** ACCORDING TO MY SOLUTION THE ANSWER IS (-1,1)


Related Questions

Berto has $12 to put gas in his car. If gas costs $3.75 per gallon, which ordered pair relating number of gallons of gas, x, to the total cost of the gas, y, includes the greatest amount of gas Berto can buy?



(__,__ )

Answers

Answer:

If gas costs $3.75 per gallon and Berto has $12, then he can purchase 12/3.75 gallons. This is approximately 3.2 gallons. So the coordinate on this line would be (3.2, 12).

Actually... It's from web

the answer is (3.2,12)

If j=h and k=m then which expression represents the value of g

Answers

Answer:

Step-by-step explanation:

as given j=h and k=m then which expression for g is

as from attached image , we can see that its look like two right angle triangle

so from lower triangle we apply the pythagorean theorem .

which state that sum of the squares on the legs of a right triangle is equal to the square on the hypotenuse.

so small triangle ΔEBD

h² +k²= g²       ---------------(1)

now from big triangle ΔABC

(j + h)² +(m + h)² =f²---------(2)

given j=h and k=m so from equation-2 we can find ,

(h + h)² +(k+ k)² =f²

(2h)² +(2k)²=f²

4h²+ 4k²= f²

4(h²+k²)= f² ----------(3)

now from equation 1 and 3 we can obtained .

4×g² =f²

take square root both side we get,

2×g=f

so g=f/2----- Answer

The Grade 5W students at the Eber Prep School wanted to create a mural on one
side of the walls in their classroom. the area of the wall is 36 square meters or 36
meters square.

Answers

Answer:

the answer is 36 square meters since this is the proper way to phrase this answer but if you do say it the other way people will now what u are talking about

11. Square ABCD shown has sides of length 10 centimeters. The unshaded
portions are both semicircles. Calculate the area of the shaded portion to
the nearest tenth of a square centimeter. *

Answers

Answer:

21cm^2

Step-by-step explanation:

Area of Square-Area of semi circles (makes one full circle)

Squares Area = 100

LxW or (10x10)=100

Circle Area = 25/pi

/pi (radius)^2 or /pi(5)^2 = 25/pi

100cm^2-25cm^2 = 21.4601…cm^2 rounded to 21.5 cm^2

The area of the shaded region is 21.5 cm²

What is a semi-circle?

'In geometry, a semicircle is a plane figure that is formed by dividing a circle into exactly two parts.'

According to the given problem,

Side length of the square = 10

Area of the square = ( 10 × 10 )

                               = 100 cm²

Now, for the semi-circle,

Diameter = 10 cm

Radius = [tex](\frac{10}{2}[/tex][tex])[/tex] cm

            = 5 cm

Area of the semicircle =  [tex]\frac{\pi r^{2} }{2}[/tex]

                                     = [tex]\frac{\pi * 5^{2} }{2}[/tex]

Since, there are two semicircles in the square, we multiply the area with 2,

⇒ [tex]\frac{\pi *5^{2} }{2}*2[/tex]

= 25[tex]\pi[/tex]

Area of the shaded region = ( 100 - 25π )

                                            = ( 100 - 78.53 )

                                            = 21.46

                                            ≈ 21.5 cm²

Hence, we can conclude, the area of the shaded region is 21.5 cm².

Learn more about semi-circles here: https://brainly.com/question/14429410

#SPJ2

700=132.69x-25.96
solve please

Answers

Answer:

x = 5.5 (rounded)

Step-by-step explanation:

Equation: 700 = 132.69x - 25.96

Add 25.96 to both sides: 700+25.96 = 132.69x -25.96 + 25.96

Simplify: 725.96 = 132.69x

Isolate x

Divided both sides by 132.69: [tex]\frac{725.96}{132.69} = \frac{132.69x}{132.69}[/tex]

Simplify: x = 5.5 (rounded)

can you find function asap please​

Answers

Answer:

i think the function is 7

Step-by-step explanation:

HELP PLEASE !!!!!!!!!!!

Answers

Answer:

option B

Step-by-step explanation:

[tex]\lim_{x\to \infty} f(x) = \lim_{x \to \infty} (log(x+2)-3 )[/tex]

                    [tex]= \lim_{x \to \infty} ( log(x+2)) - \lim_{x \to \infty} 3\\\\=\infty - 3\\\\= \infty[/tex]

[tex]\lim_{x \to 2} f(x) = \lim_{x \to 2} ( log(x+2)) - \lim_{x \to2} 3\\\\[/tex]

                 [tex]= log( 2 + 2 ) - 3\\\\= log 4 - 3\\\\=log(2^2) -3\\\\=2 log 2 -3[/tex]

[tex]\lim_{x \to -2} f(x) = \lim_{x \to -2} ( log(x+2)) - \lim_{x \to -2} 3\\\\[/tex]  [tex]\lim_{x \to -2}_+ f(x) = \lim_{x \to -2_+} ( log(x+2)) - \lim_{x \to -2_+} 3 = -\infty\\\\\lim_{x \to -2_-} f(x) = \lim_{x \to -2}_- ( log(x+2)) - \lim_{x \to -2}_- 3 = -\infty\\\\\\\lim_{x \to -2} f(x) = - \infty[/tex]

Which quotient is greater than 3? 28 divided by 10, 35 divided by 11, 38 divided by 13 or 40 divided by 14

Answers

Simply divide these all! 28/10 is 2.8, 35/11 is 3.1, 38/13 is 2.9, and 40/14 is 2.8! 35/11 is the only one that is more than 3

The quotient 35 divided by 11 is the only one among the options that is greater than 3.

To determine which quotient is greater than 3, we can calculate the value of each quotient:

28 divided by 10 = 2.8

35 divided by 11 ≈ 3.182

38 divided by 13 ≈ 2.923

40 divided by 14 ≈ 2.857

Comparing these values, we can see that only the quotient 35 divided by 11, which is approximately 3.182, is greater than 3.

Therefore, the quotient 35 divided by 11 is the only one among the options that is greater than 3.

To know more about divided:

https://brainly.com/question/15381501


#SPJ6

Why is it easy to hammer a sharp pin than a blunt pin ? *

Answers

Answer:

It is easier to hammer a sharp nail than a blunt one

Step-by-step explanation:

This is because blunt nail occupies more area than sharped nail. As sharp will exert more pressure as it occupies less area.

Ms. Lesure is going to buy lunch for Mr. Shows Interactions A class. How many whole pizzas would she need to buy (assume each pizza has 8 slices) assuming there are 10 boys and 8 girls and the boys will eat 2 slices each and the girls will eat 1.5 slices each?

Answers

Answer:

Ms. Lesure is going to need to buy 4 pizzas for Mr. Shows Interactions A class. The boys would eat 20 slices and the Girls in total will eat 12 slices. In total you'll need about 32 slices and 32 divided by 8 is 4.

Step-by-step explanation:

Can anyone help me with this please? I need help along with an explanation.

Answers

the value of a/b = 2.5 because a = 5 and b = 2.

if a^2 - b^2 = 21,
5^2 = 25, 2^2 = 4,
25 - 4 = 21

if a - b = 3,
5 - 2 = 3

5/2 = 2.5

hope this helps! :)

Answer:

a + b = 7

Step-by-step explanation:

a^2 - b^2 = 21

Factor the left side. It is the difference of two squares which factors into the product of a sum and a difference.

(a + b)(a - b) = 21

We are told a - b = 3, so substitute 3 for the factor a - b.

(a + b)(3) = 21

Divide both sides by 3.

a + b = 7

Solve for w.
69=5w – 16
Simplify your answer as much as possible.

Answers

w=17 im too good at math to be wrong about this
69 = 5w - 16

69 + 16 = 5w

85 = 5w

Divide both sides by 5.

W = 17

Bertie won the lottery! He spends 4/5 of his money on a large house. He spends 3/4 of his REMAINING money on a luxury yacht. He spends 2/3 of his remaning money on a trip to space. He gives the remaning $50,000 to a charity. How much did he win?

Answers

Answer:

Step-by-step explanation:

Step 1

Find the fraction of the money he gave the charity

Let the total amount he had = 1

Bertie won the lottery! He spends 4/5 of his money on a large house. He spends 3/4 of his REMAINING money on a luxury yacht. He spends 2/3 of his remaning money on a trip to space.

Fraction of money he gave to charity =

1 - (4/5 + 3/4 + 2/3)

He gives the remaning $50,000 to a charity. How much did he win?

Answer pls…………………………..

Answers

Answer:

Option 3

Step-by-step explanation:

Brainliest please~

please help soon, i can't seem to figure this one out!!

Consider the following equation. -2x + 6 = -(2/3)^x + 5. Approximate the solution to the equation above using three iterations of successive approximation. Use the graph below as a starting point.

A. x = 3/4
B. x = 13/16
C. x = 7/8
D. x = 15/16

Answers

I think the answer is B

The required solution after three successive iterations is near x = 7/8. Option c is correct.

Equation is -2x + 6 = -(2/3)^x + 5. Approximate the solution to the equation above using three iterations of successive approximation.

What is the equation?

The equation is the values of two expressions that are equal.

Here,

[tex]-2x + 6 = -(2/3)^x + 5[/tex]

Arranging the equation

[tex]-2x + 6 +(2/3)^x - 5 = 0\\(2/3)^x-2x+1=0[/tex]
Since the solution of the equation is when f(x) = 0 at near to  x = 0.8
[tex]f(x) = (2/3)^x-2x+1\\[/tex]

First iteration at x =0.8
[tex]f(0.8) = (2/3)^x-2x+1\\ = (2/3)^{0.8}-2*0.8+1\\ = 0.123[/tex]

It will go up more to get exact zero, so
Second iteration at x = 0.9

[tex]f(0.9) = (2/3)^{0.9}-2*0.9+1\\f(0.9) = -0.106[/tex]

It seems that zero is near 0.9 so will go down
The third iteration at x = 0.87

[tex]f(0.87) = (2/3)^{0.87}-2*0.87+1\\f(0.87) = -0.037[/tex]

Here , solution is much near to x = 0.87 or x =7/8.


Thus, the required solution after three successive iterations is near x = 7/8.

Learn more about the equation here:
https://brainly.com/question/10413253

#SPJ5

(3a+b)³ please help me​

Answers

Answer:

[tex]27a^{3} +27a^{2}b+9ab+b^{3}[/tex]

Step-by-step explanation:

[tex](3a+b)(3a+b) = 9a^{2}+6ab+b^{2}\\(9a^{2}+6ab+b^{2})(3a+b) = 27a^{3} + 27a^{2}b +9ab^{2} +b^{3}[/tex]

Answer:

Step-by-step explanation:

9a^3+18a^2b+9ab^2+b^2

Simplify the expression. x^5/x^9

Answers

Answer:

1/x^4

Step-by-step explanation:

Find the sale price of an $18 item after a 50% discount.

Answers

Rewrite 50% as a decimal : 0.50

Multiply the price by 0.50:

18 x 0.50 = 9

The sale price is $9

Answer:

$9

Step-by-step explanation:

price of an item=$18

discount=50%

sale price =? (be x)

sale price= original price -discount% of original price

x=$18 -50/100 * $18

x=$1800-$900/100

=$900/100

=$9

therefore sale price of an item is $9.

can someone help i dont understand exponential functions

Answers

Answer:

1,3,4,6

Step-by-step explanation:

13 14 14 17 18 20 20 what's the mode?

Answers

Answer:

mode is 14,20

if you dont know mode is the frequency of the numbers

Step-by-step explanation:

Answer:

14 & 20

Step-by-step explanation:

the number that appears the most

Middle School teaches 6th, 7th, and 8th grade classes. The sixth grade has 318 students, the 7th has 286 students, and the 8th has 306 students. If the sixth grade takes a field trip and each student needs $3.50, how much total money is needed by the students? $910 c. $1113 b. $3185 d. $1071​

Answers

Answer:

$3185

Step-by-step explanation:

Multiply the number of grade 6 students by the money needed ($3.50) and you get 318x3.50=1113

Multiply the number of grade 7 students by the money needed ($3.50) and you get  286x3.50=1001

Multiply the number of grade 8 students by the money needed ($3.50) and you get 306x3.50=1071

Add all the total amounts together and you get $3185

the slope of the line that passes through the points in the table is:

Answers

Answer:

-2/7

Step-by-step explanation:

We have two points so we can use the slope formula

m = (y2-y1)/(x2-x1)

m = (8-0)/(-14-14)

   = 8/-28

   = -2/7

7.
A ball is thrown into the air with an upward velocity of 36 ft/s. Its height h in feet after t seconds is given by the function h=-16t^2+36t+9. In how many seconds does the ball reach its maximum height? Round to the nearest hundredth if necessary. What is the ball’s maximum height?

A. 1.13 s; 69.75 ft

B. 1.13 s; 29.25 ft

C. 2.25 s; 9 ft

D. 1.13 s; 31.5 ft

Answers

Answer:

The answer is 1.13;29.25

Step-by-step explanation:

Answer: The ball reaches its maximum height in 1.13 seconds. b) The ball's maximum height is 30.25 feet.

This sort of problem is solved easily by a graphing calculator.

a) The ball reaches its maximum height in 1.13 seconds.

b) The ball's maximum height is 30.25 feet.

_____

Since you want to know when and where the peak value of the function occurs, it is convenient to put it into vertex form.

 h = -16(t² -(9/4)t) + 10

 h = -16(t² -(9/4)t +(9/8)²) +10 +16(9/8)²

 h = -16(t -9/8)² + 10 + 81/4

 h = -16(t -9/8)² + 30 1/4

The vertex of the function is (9/8, 30 1/4) ≈ (1.13, 30.25).

h = -16t^2 + 36t + 9

At maximum height h' = 0,

h' = -32t + 36 = 0

32t = 36

t = 36/32 = 1.13 s.

Maximum height = -16(1.125)^2 + 36(1.125) + 9 = 29.25 ft

Answer is option d (1.13 s, 29.25 ft)

The answer is 1.13;29.25

What is the following product?
3/24 • 3/45

Answers

Answer:

6(³√5)

Step-by-step explanation:

³√24×45=6(³√5)

hope my answer helps!

Julie is solving the equation x2 + 5x+6= 0 and notices that the discriminant b2- 4ac has a value of 1. This tells her that the equation
has
A) no real roots

B) exactly one real root.

C) exactly two real roots.

D) exactly three real roots.

Answers

Answer: C

Step-by-step explanation:

If you use the quadratic equation you will find that there are two possible answers to this problem

The explanation shows that the equation has exactly two real roots. so the correct option is C.

What is a quadratic equation?

A quadratic equation is the second-order degree algebraic expression in a variable. the standard form of this expression is  ax² + bx + c = 0 where a. b are coefficients and x is the variable and c is a constant.

Given that Julie is solving the equation x² + 5x+6= 0 and notices that the discriminant b²- 4ac has a value of 1.

x² + 5x+6= 0

The discriminant = b²- 4ac = 1

It will be Real and distinct because the discriminant is positive.

There are two roots of a quadratic equations always if  the discriminant is positive. We say that the quadratic equation has 2 solution if roots are distinct, and have 1 solutions when both roots are same.

Hence, This tells her that the equation has exactly two real roots. so the correct option is C.

Learn more about quadratic equations;

https://brainly.com/question/17177510

#SPJ2

Someone help me with this……

Answers

Answer:

its the second won

Step-by-step explanation:

The second one is the correct answer.

A store marks up a $550 widescreen television by 30%. During a sale it is discounted by 20%. What is the percentage change in the original price of the television?

Answers

Answer:

4%

Step-by-step explanation:

Marked Price = 130% of cp

= 1.3 x 550

= 715

Selling price = 80% of mp

= 0.8 x 715

= 572

% change in price =

[tex] \frac{572 - 550}{550} \times 100 = 4[/tex]

4%

What is 300+3x40-25x4=?

Answers

Answer:

320

Step-by-step explanation:

300+3x40=120-25x4

300+120-25x4=100

320

Answer:

300+3x40-25x4 = 320

Step-by-step explanation:

300+3x40-25x4

= 300+ 120 - 100

= 420-100

= 320


Which of the following could be the measure of the angle below?

Answers

Answer:

There is nothing below.

Step-by-step explanation:


[tex] {( \frac{5}{3} )}^{2n + 1} {( \frac{5}{3} )}^{5} = ({ \frac{5}{3} )}^{n + 2} [/tex]
Pls include steps...

Answers

Answer:

n = -4

Step-by-step explanation:

[tex](\frac{5}{3} )^2^n^+^1 * (\frac{5}{3} )^5 = (\frac{5}{3})^n^+^2[/tex]

in multiplication if the base are same u can add there exponent.

[tex](\frac{5}{3})^2^n^+^1^+^5 = (\frac{5}{3})^n^+^2[/tex]

base of both sides are equal so their exponent will be equal

2n + 1 + 5 = n + 2

2n + 6 = n + 2

2n - n = 2 - 6

n = -4

Other Questions
What is the uses of seawater as a source of renewable energy? If a normal distribution has a mean of 104 and a standard deviation of 6, what is the z-score for a value of 110?A. 1.5B. 2C. 1D. 0.5 What is the volume of the cone in the diagram? Which long term effects did the industrial revolution eventually have on the lives of industrial worker's HELPPPPPPPPPPPPPPPPPP ASAP!!!!!!!!!!!!!!!! In the equation below assume U and A are know.Rearrange the equation so that you would solve for R in terms of the Greek letter sigma)U = R 1 2 A 2A. R = U 1 2 A 2B. R = 1 2 A 2 UC. R = U 1 2 A 2 You send a traveling wave along a particular string by oscillating one end. If you increase the frequency of oscillations, does the speed of the wave increase, decrease, or remain the same? what is the digiMelt Start Temp and the digiMelt Stop Temp for Formic Acid Acetic Acid Propionic Acid What is x if x + 2y = 10 and y = 3? The measure of arc XY is 85 degrees. The measure of arc WZ is 25 degrees. What is the measure of angle 3? The diagram shows an ocean food web. Which organism is a primary consumer?krillelephant seal squidpenguin Does this table represent a function? Why or why not? A father who was paralyzed in an accident and unable to work refused to accept financial support from relatives or the government to help him care for his family. When one of his children became seriously ill and needed medical attention, the father refused to allow the child to receive medical care at the local public health clinic because of his feelings regarding the acceptance of charity. The father thought that the child was starting to get better, but then she took a turn for the worse and died from her illness. What crime has the father most likely committed Which of these answer choices is true about the pattern Nancy ate 34 ounces of cake. How much cake did she eat? A. 1.500 lb. B. 2.125 lb. C. 2.500 lb. D. 2.750 lb.Nancy ate 34 ounces of cake. How much cake did she eat? A. 1.500 lb. B. 2.125 lb. C. 2.500 lb. D. 2.750 lb. Read the paragraph, which is a summary of a slave narrative by Olaudah Equiano.[1] While he was working for Thomas Farmer, he began to buy fruits and other goods and sell them himself. [2] He faced insults and difficulties doing this; however, he was determined to continue making money, which he did. [3] He wanted to obtain enough savings to buy his freedom, which Robert King had promised him he could do for the same amount that King had paid for him. [4] When he had saved enough money, he bought his freedom from King and lived the rest of his life as a free man.Which sentence in the paragraph is structured differently than the others? eeeasdaaaa asdsaa asdsaaa asdsaaa asdsaaa asdsaaa asdsaaa asdsaaa asdsaaa asdsaaa asdsaa Please help me help with question 3, 4, and 5 please help What are 3 artificial and 2 natural sources of electromagnetic radiation? ANSWER ASAP PLEASEAccording to this table, which star has the highest absolute brightness? a. Betelgeuseb. Rigelc. Bellatrix